PT5.S1.Q15 - People who primarily use their left hand

Ali MarieAli Marie Member
edited February 2021 in Logical Reasoning 81 karma

Hi. Can anyone explain why E is incorrect and how A is supported? Thanks!!

Admin Note: I deleted the question and answer choices as it is against our Forum Rules to post LSAT questions and answer choices on the Forum.

Comments

  • LogicianLogician Alum Member Sage
    2464 karma

    Hey there,

    This question stem is definitely an odd one and also a bit outdated, but essentially the question stem is asking which one of these answer choices could M's evidence support (if even the slightest), and subsequently be countered by Q's (sorry if i'm being redundant, just doing it for the sake of clarity)

    So M's evidence is that its nearly impossible to find a person between the ages of 85-90 who's left handed. So with this evidence, what could M possibly postulate? well maybe M thinks that because they're are almost no old lefties (they're almost all righties), there may be some sort of advantage related to being right handed, presumably that you live longer. Though it may not be apparent at first, this is a classic case of correlation/causation. But just because nearly all 85-90 year olds are right handed, does that mean that being right handed somehow makes you live longer? of course not- which is precisely the role of Q's evidence; the 85-90 year olds of today were forced to be right handed 70-90 years ago. This explains away M's causal hypothesis- its not that being right handed makes you live longer, its just that everyone has been forced to be right handed, hence why you cant find any 85-90 year olds who are left handed.

    AC E says: "physical habits learned in school often persist in old age"
    So first off, what are the physical habits learned in school? is it raising your hand when you have a question? or is it what hand you write with? In any case, lets be generous and assume were talking specifically about what hand you write with, so "whatever hand you learn to write with in school you continue to write with in old age". Now, applying this to the M's evidence we get:

    M's Hypothesis: Whatever hand you learn to write with in school you continue to write with in old age.
    M's evidence: It is nearly impossible to find a person between 85-90 who's primarily left handed.

    Q's counter: 70-90 years ago left handed children were forced to use their right hands.

    So by applying AC E Q's evidence would not serve as a counter, but rather just as additional information saying "yeah 70-90 ago, kids were forced to learn to write with their left hands"

    hope this helps!

  • Ali MarieAli Marie Member
    81 karma

    Extremely helpful!! Thank you very much :)

  • Burden.of.FloofBurden.of.Floof Core Member
    1050 karma

    What is a good way to parse out the grammar with certainty from this question stem?

    Now I see that we're trying to pick a hypothesis for M which makes the right AC a lot clearer, but when I was answering the question I thought it meant we were supposed to pick a hypothesis for Q.

  • canihazJDcanihazJD Alum Member Sage
    edited June 2021 8460 karma

    @"Burden.of.Floof"

    Q's response serves to counter any use by M of the evidence about 85 to 90 year olds own support of which one of the following hypotheses?

    Q's response serves to counters any use by M of the evidence about 85 to 90 year olds own supporting of which one of the following hypotheses?

    or

    Which conclusion did Q prevent M from drawing?

  • Burden.of.FloofBurden.of.Floof Core Member
    1050 karma

    aaah okay got it. Thanks @canihazJD!

Sign In or Register to comment.